0 Daumen
1k Aufrufe

Fkt.: ax^3-3ax+1

davon muss ich die extrempunkte berechnen

also ich habe mit den ableitungen angefangen:

f'(x)=3ax^2-3a

f''(x)=6ax

f'''(x)=6a

danach: notw. Bed: f'(x)=0

*und hier bin ich mir nicht mehr so sicher*

3ax^2-3a=0 | +3a

3ax^2=3a | :3a (ich bin mir nicht sicher, ob ich das machen darf)

x^2=a | wurzel

x=Wurzel a

hin. Bed. : f'(x)=0 und f''(x) =/=0

f''(wurzel a) = 6 * Wurzel a * x >0=> Tiefpunkt

f"(-wurzel a) = 6+ (-(wurzel a))*x <0=> Hochpunkt ( wenn man die wurzel zieht kann man ja auch negative werte rausbekommen, deswegen hier auch einen Hochpunkt

Ist es richtig soweit? Wenn nicht, wie soll ich anders machen? Ich bin mir halt nicht sicher, da das Ganze komisch aussieht mit dem a und mit der Wurzel

Avatar von

2 Antworten

+1 Daumen
 
Beste Antwort

Hallo,

:3a (ich bin mir nicht sicher, ob ich das machen darf) --->JA

y = a x^3 -3ax +1

y' =3a x^2 -3a =0 |+3a

3a x^2  = 3a  | :3a     a≠0

x^2= 1

x1.2= ± 1

blob.png

Avatar von 121 k 🚀

achso es hat also soweit gestimmt nur, dass 3a : 3a = 1 und nicht a
jetzt sieht es auch viel besser aus

danke :)

so ist es :)

0 Daumen

3ax^2-3a=0 | +3a

3ax^2=3a | :3a (ich bin mir nicht sicher, ob ich das machen darf)

x^2=a | wurzel

Wenn a=0 ist, wird f(x) zu f(x)=1, da die Terme mit x wegfallen. Daher darf durch 3a geteilt werden. Dein Fehler steckt in der nächsten Zeile:  (3a)/(3a)=1

Also: \(x^2=1 \Rightarrow x_{12}=\pm 1\)


Avatar von 47 k

Ein anderes Problem?

Stell deine Frage

Willkommen bei der Mathelounge! Stell deine Frage einfach und kostenlos

x
Made by a lovely community